0 Daumen
3,7k Aufrufe

Stimmt der folgende Beweis?

Beweis, dass \(\left(\frac{(-1)^n}{\sqrt{n}}\right)_{n\geq 1}\) eine Nullfolge ist.

Da in der Vorlesung gezeigt wurde, dass \(\frac{1}{n}\) eine Nullfolge ist, habe ich so umgeformt:

Für alle  \(n \in \mathbb{N}\) gilt \(\left\lvert\frac{(-1)^n}{\sqrt n}\right\rvert=\frac1{\sqrt n}\). Dann ist \(\lim\limits_{n\to\infty}\frac{1}{\sqrt{n}}=\left[\lim\limits_{n\to\infty}\left(\frac{1}{n}\right) \right]^{\frac{1}{2}}=0^{\frac{1}{2}}=0\).

Wie würde man das mit der Epsilontik machen?

Definition:

Für jedes \(\varepsilon >0\) exisitiert ein \(N\in \mathbb{N}\) derat, dass \(|z_n-z|< \varepsilon\) für \(n\geq N\), d. h. \(z_n \in B_\varepsilon(z)\) für alle \(n\geq N\).

Ich komme hier ab \(\left\lvert\frac{(-1)^n}{\sqrt n}\right\rvert= \frac1{\sqrt n}< \varepsilon\) nicht mehr weiter...

Avatar von 28 k

Hi!

Als Tipp vom Tutor kam: "Ihr könnt bei dieser Aufgabe alle anderen Aufgaben verwenden,
die ihr auf diesem Blatt zeigen müsst - insbesondere Aufgabe 2 kann euch weiterhelfen."

Danach könntest du die Aussage 2iii als bewiesen voraussetzen. Dann könntest du die Folge in zwei folgen aufteilen $$(-1)^n$$ und diese Folge ist nach Bemerkung 7.7. 2. beschränkt. jetzt musst du nur noch zeigen, dass die Folge $$\frac{1}{\sqrt{n}}$$ eine Nullfolge ist.

Ich habe das via Epsilon-Delta-Kriterium gemacht und zum Erfolg gelangt. Hier kann man ja noch leicht umstellen, aber bei Ausdrücken wie \(\left(\frac{n!}{n^n}\right)_n\) hört das ganze auch wieder auf, habe hier in die Produktschreibweise umgeschrieben...$$\frac{n!}{n^n}=\frac{\prod_{k=1}^nk}{\prod_{k=1}^nn}=\prod_{k=1}^n\frac kn=\frac1n\cdot \prod_{k=2}^n\frac kn$$

Aber reicht meins nicht auch bereits? Ich meine 1/n ist eine Nullfolge und es gilt \(\frac1n\cdot \prod_{k=2}^n\frac kn\le \frac1n\)

$$\left(\frac{1}{n^2}+\frac{2}{n^2}+ ... +\frac{n}{n^2}\right)_{n\ge1}$$ kannst du auch schreiben als

$$\left(\frac{1+2+3+4+...+n}{n^2}\right)_{n\ge1}$$ der Zähler ist $$\sum_{k=1}^{n}{k}=\frac{n(n+1)}{2}$$ im Skript Beispiel 3.9.

Dann kannst du schreiben $$ \left(\frac{\frac{n(n+1)}{2}}{n^2}\right)_{n\ge1}$$

Ich bin müde aber wenn ich jetzt nicht völlig daneben bin sollte das jetzt $$\left(\frac{1}{2}+\frac{1}{2n}\right)_{n\ge1}$$ gegen $$\frac{1}{2}$$ konvergieren.

Yeah, stimmt — danke. LG

Zu deinem vorherigen Kommentar: nach Definition 7.5. im Skript ist n! nicht beschränkt. Also kannst du 2 iii) nicht anwenden. Jetzt muss ich ins Bett.

1 Antwort

0 Daumen
 
Beste Antwort

Hallo

 1/√n<ε wenn 1/n<ε^2 oder n>=N=[1/ε^2]

 Man rechnet  N(ε) meist einfach aus während du erst beweisen müsstest dass man lim und Wurzel einfach so vertauschen darf. Dein Beweis ist also nicht gut.

Gruß lul

Avatar von 106 k 🚀

Aber das Quadrieren ist doch keine Äquivalenzumformung, oder?

Hallo

 sowohl n, wie ε, wie √n sind >0, also  kann man jeden Schritt auch rückwärts gehen,  (muss man aber nicht können) du fängst ja an mit sei ε beliebig >0 usw. für positive Zahlen ist alles ok. (du kannst ja auch mit ε1=ε^2>0 anfangen.)

Gruß lul

Ah, stimmt. Ja gut.

Also kann ich sagen, dass für \(n>\frac{1}{\varepsilon ^2}=N\) nun \(\bigg |\frac{(-1)^n}{\sqrt{n}} \bigg | < \varepsilon\) gilt? Und warum?

Ein anderes Problem?

Stell deine Frage

Willkommen bei der Mathelounge! Stell deine Frage einfach und kostenlos

x
Made by a lovely community